每日一题[560]方程组的解

关于$x,y$的方程组$\begin{cases} x^2+y^3=29,\\ {\log_3}x\cdot {\log_2}y=1\end{cases} $的不同实数解的组数是______.


cover

分析与解 题中方程组可以变形为$$\begin{cases} x^2+y^3=29,\\ {\log_3}x^2\cdot {\log_2}y^3=6,\end{cases} $$于是问题可以转化为求关于$x,y$的方程组$$\begin{cases} x+y=29,\\ \ln x \cdot \ln y=6\ln 2\cdot \ln 3,\end{cases} $$的实数解组数,即关于$x$的方程$$\ln x\cdot \ln (29-x)=6\ln 2\cdot \ln 3$$的实数解个数.

显然,有$1<x<28$,令$f(x)=\ln x\cdot \ln (29-x)$($x\in (1,28)$),则其导函数$$f'(x)=\dfrac{(29-x)\ln (29-x)-x\ln x}{x(29-x)},1<x<28,$$考虑到函数$y= x\ln x$在$(1,28)$上单调递增,于是函数$y=(29-x)\ln (29-x)-x\ln x,1<x<28$单调递减,在区间$(1,28)$上有唯一零点$x=\dfrac {29}2$.因而函数$f(x)$在$\left(1,\dfrac{29}2\right)$上单调递增,在$\left(\dfrac{29}2,28\right)$上单调递减,考虑到$$f(1)=0,f\left(\dfrac{29}2\right)=\ln ^2\dfrac{29}2> \ln 8\cdot \ln 9=6\ln 2\cdot \ln 3,f(28)=0,$$因此关于$x$的方程$$\ln x\cdot \ln (29-x)=6\ln 2\cdot \ln 3$$的实数解个数为$2$.

综上所述,题中方程组的不同实数解的组数为$2$.

 先将$x$的取值范围加以限定以避开函数$y=x\ln x$的单调递减区间,减少说理的难度.

此条目发表在每日一题分类目录,贴了标签。将固定链接加入收藏夹。

发表回复